Difference between revisions of "2000 AMC 10 Problems/Problem 6"

(Problem)
(duplicate)
 
Line 1: Line 1:
==Problem==
+
#REDIRECT [[2000 AMC 12 Problems/Problem 4]]
 
 
The Fibonacci sequence <math>1, 1, 2, 3, 5, 8, 13, 21, \ldots</math> starts with two <math>1</math>s, and each term afterwards is the sum of its two predecessors. Which one of the ten digits is the last to appear in the units position of a number in the Fibonacci sequence?
 
 
 
<math>\mathrm{(A)}\ 0 \qquad\mathrm{(B)}\ 4 \qquad\mathrm{(C)}\ 6 \qquad\mathrm{(D)}\ 7\qquad\mathrm{(E)}\ 9</math>
 
 
 
==Solution==
 
 
 
The pattern of the units digits are
 
 
 
<math>1,1,2,3,5,8,3,1,4,5,9,4,3,7,0,7,7,4,1,5,6</math>
 
 
 
In order of appearance:
 
 
 
<math>1,2,3,5,8,4,9,7,0,6</math>.
 
 
 
<math>6</math> is the last.
 
 
 
<math>\boxed{\text{C}}</math>
 
 
 
==See Also==
 
 
 
{{AMC10 box|year=2000|num-b=5|num-a=7}}
 

Latest revision as of 23:34, 26 November 2011